Вы находитесь на странице: 1из 70

TEARS OF OLD PM ------>>>

T - Territory ,
E - Emoluments,
A - Affirmations/Oaths,
R - Rajya Sabha,
S - Scheduled Areas,

O - Other Scheduled Areas (Meghalaya, Tripura, Mizoram and Assam) - Remember it considering
you MeT MiA (famous Vj),
F - Federal structure - Three Lists,

O - Official Languages,

L - Land Reforms - 9th Schedule


D - Defection -- Anti- Defection,

P - Panchayats,

M - Municipalities
WE, THE PEOPLE OF INDIA, have solemnly resolved to constitute India into a
SOVEREIGN SOCIALIST SECULAR DEMOCRATIC REPUBLIC and to secure to all its citizens:
JUSTICE, social, economic and political;
LIBERTY of thought, expression, belief, faith and worship;
EQUALITY of status and of opportunity;
and to promote among them all
FRATERNITY assuring the dignity of the individual and the unity and integrityof the Nation;
IN OUR CONSTITUENT ASSEMBLY this twenty-sixth day of November, 1949, do HERE BY ADOPT,
ENACT AND GIVE TO OURSELVES THIS CONSTITUTION.

, ,
[1] ,
:
, , , , ,
, ,
,
,
[2]
, ,
26 1949
( , )
,

.
U Can Fly Directly From US to UP to Meet Sita Ram
Free Transport Service To Every Senior & Older
Even Mahila Attains Transport Services

Part Subject Articles


Part I The Union and its territory Art. 1 to 4
Part II Citizenship Art. 5 to 11
Part III Fundamental Rights Art. 12 to 35
Part IV Directive Principles Art. 36 to 51
Part IVA Fundamental Duties Art. 51A
Part V The Union Art. 52 to 151
Part VI The States Art. 152 to 237
Part VII Repealed by Const. (7th Amendment) Act, 1956
Part VIII The Union Territories Art. 239 to 242
Part IX The Panchayats Art. 243 to 243O
Part IXA The Muncipalities Art. 243P to 243ZG
Art. 243ZH to
Part IXB The Co-operative Societies
243ZT
Part X The Scheduled and Tribal Areas Art. 244 to 244A
Part XI Relations between the Union and the States Art. 245 to 263
Part XII Finance, Property, Contracts and Suits Art. 264 to 300A
Part XIII Trade, Commerce and Intercourse within the Territory of India Art. 301 to 307
Part XIV Services under the Union and the States Art. 308 to 323
Part
Tribunals Art. 323A to 323B
XIVA
Part XV Elections Art. 324 to 329A
Part XVI Special provisions relating to certain classes Art. 330 to 342
Part XVII Official Language Art. 343 to 351
Part Subject Articles
Part XVIII Emergency Provisions Art. 352 to 360
Part XIX Miscellaneous Art. 361 to 367
Part XX Amendment of the Constitution Art. 368
Part XXI Temporary, Transitional and Special Provisions Art. 369 to 392
Short title, commencement, authoritative text in Hindi and
Part XXII Art. 393 to 395
repeals

MCQs on Preamble of Indian Constitution - 1


1. The first country who adopted Preamble for its written constitution
(a) USA
(b) India
(c) Britain
(d) Canada

2. The Preamble of Indian Constitution is based on


(a) Philosophy of India
(b) Objective Resolution
(c) Indian Culture
(d) Religious Concept

3. At which part of constitution declared that India is a SOVEREIGN SOCIALIST


SECULAR DEMOCRATIC REPUBLIC
(a) Fundamental Right
(b) Directive Principles of State Policy
(c) Preamble
(d) Fundamental Duties

4. The term 'We' in preamble means


(a) Indian Government
(b) Supreme Court
(c) Indian Parliament
(d) The People of India

5. The order of following words seen in Preamble is


1. DEMOCRATIC
2. SOCIALIST
3. SOVEREIGN
4. SECULAR
5. REPUBLIC
(a) 3, 2, 4, 1, 5
(b) 2, 3, 4, 1, 5
(c) 3, 2, 1, 4, 5
(d) 3, 1, 2, 5, 4

6. The ideal of justice - social, economic and political- mentioned in the preamble
has been taken from
(a) French Revolution
(b) Russian Revolution
(c) American Civil War
(d) None of the above

7.The three words - liberty, equality and fraternity- mentioned in our Preamble have
been taken from
(a) Russian revolution
(b) American Civil War
(c) French Revolution
(d) None of the above

8. Indian Constitution ensure 'Justice' in which of the following


(a) Social
(b) Economic
(c) Political
(d) All of the above

9. At which case,the Supreme Court specifically opined that Preamble is nota part of
the Constitution
(a) Berubari Union case (1960)
(b) Kesavananda Bharati case (1973)
(c) Both (a) & (b)
(d) None of the above
10. At which case,the Supreme Court rejected the earlier opinion and held that
Preamble is an integral part of the Constitution
(a) Berubari Union case (1960)
(b) Kesavananda Bharati case (1973)
(c) Both (a) & (b)
(d) None of the above

11. Preamble has been amended by which amendment act


(a) 27 th
(b) 44 th
(c) 42 nd
(d) 40 th

12. 42nd amendment of Indian Constitution (1976) added which of the terms to
Preamble
(a) Socialist
(b) Secular
(c) Sovereign
(d) Both 1 & 2

13. Which of the following is 'TRUE' about Preamble


(a) It is a source of power to legislature and a source of prohibition upon the powers of
legislature
(b)It is Justiciable
(c) Its provisions are enforceable in courts of law
(d) None of the above

14. As per preamble date of adoption of the Constitution was


(a) 26 January 1950
(b) 26 November 1949
(c) 11 December 1946
(d) None of the above

15. The Preamble states that the Constitution derives its authority from
(a) Indian Culture
(b) Government of India
(c) The People of India
(d) Princely States

16. Which of the following is described as the Soul of the Constitution?


(a) Fundamental Right
(b) Fundamental Duties
(c) Directive Principles for State Policy
(d) Preamble

1. When was the ConstituentAssembly convened ?

Answer: 1946
2. When was the first session of theConstituent Assembly held ?
Answer: 9 December, 1946
3. Under which Plan was the Constituent Assembly constituted ?
Answer: Cabinet Mission Plan
4. Who was the President of the Constituent Assembly ?
Answer: Dr. Rajandra Prasad
5. Who was the Chairman of the Drafting Committee ?
Answer: B R. Ambedkar
6. When was the drafting of the Indian Constitution completed and adopted ?
Answer: 26 November, 1949
7. When was the Constitution of India put into effect ?
Answer: 26 January ,1950
8. Since when India became a Republic ?
Answer:26 January, 1950
9. Who was the first President of the India ?
Answer: Dr. Rajendra Prasad
10. Who was the first Vice- President of the Indian Union ?
Answer: Radhakrishnan
11. Who was the first Prime Minister of the Indian Union ?
Answer: Jawaharlal Nehru
12. Who was the last Governor- General (Viceroy) of British India ?
Answer: Lord Mountbatten
13. Who was the first Governor- General of Independent India ?
Answer: Lord Mountbatten
14. When adopted the National Flag of India ?
Answer: July 21st 1947 by The Constituent Assembly
15. Who set forth the main objectives of the Constituent Assembly in the form of the
Objective Resolution ?
Answer: Jawaharlal Nehru
16. How many articles are there in the Indian Constitution ?
Answer: 448 Articles
17. How many Fundamental Rights have been guaranteed by the Indian Constitution
?
Answer: 6(six)
18. How many Fundamental Duties have been assigned to the citizens of India ?
Answer: 10(ten)
19. Who appoints the Prime minister of India ?
Answer: The President of India
20. How is the President of India elected ?
Answer: Every elected member of the legislative assembly of a State shall have as many votes as
there are multiples of one thousand in the quotient obtained by dividing the population of the
state by the total number of elected members of the Assembly;
21. What is the name of the Upper House of the Indian Parliament?
Answer: Rajya Sabha
22. What is the name of the Lower House of the Indian Parliament?
Answer: Lok Sabha
23. Who presides over the session of the Lok Sabha?
Answer: The Speaker
24. Who presides over the session of the Rajya Sabha?
Answer: The Vice-President of India
25. which article give special status to Jammu & Kashmir ?
Answer: Article 370
26. Who appoints the Governors of States?
Answer: The President of India
27. What is the total number of members in the Lok Sabha?
Answer: 552
28. What is the number of elected members in the Lok Sabha?
Answer: 530
29. How many members constitute the Rajya Sabha?
Answer: 250
30. How are the members of the Rajya Sabha elected?
Answer: Membership is limited to 250 members, 12 of whom are nominated by the President of
India for their contributions to art, literature, science, and social services.
31. who designed rupee symbol?
Answer: symbols adoption in July 2010 by D Udaya Kumar
32. who designed Indian flag ?
Answer: The Indian national flag was designed in 1916 by Pingali Venkayya from Vijayawada
33. What is the tenure of Lok Sabha?
Answer: 5 years
34. Which is the highest Judicial Court of India?
Answer: The Supreme Court
35. How many Judges constitute the Supreme Court?
Answer: 30 Judges
36. How many times till 2013 the Parliamentary Elections have been held?
Answer: 15 times
37. A citizen of how many years of age can cast vote?
Answer: 18 years
38. How many Indian Territories are there in the Indian Union?
Answer: 7 (seven)
39. How many times (January 2013) Indian Constitution has been amended?
Answer: 98 times
40. How many high court in India
Answer: India's unitary judicial system is made up of the Supreme Court of India at the national
level, for the entire country and the 24 High Courts at the State & Union territory level.
100 MOST IMPORTANT QUESTION BASED ON THE INDIAN CONSTITUTION FR THE
COMPETITIVE EXAMS***

1. Through which constitutional amendment in article 359, it has been laid down that Fundamental
Rights under articles 20 and 21 are enforceable during the operation of emergency.
(A) 44th Amendment Act SSC General Knowledge : The Constitution of India
(B) 46th Amendment Act
(C) 45th Amendment Act
(D) 48th Amendment Act

2. On whose satisfaction period of emergency shall be extended for operation in case security of
India or any part of the Indian territory is threatened.
(A) Prime Minister
(B) Home Minister
(C) President of India
(D) Vice-President of India

3. Article 20 of the Fundamental Rights represents which subject.


(A) Equality of opportunity in matters of public employment
(B) Protection in respect of Conviction of Offence
(C) Protection of life and personal liberty
(D) None of the above .

4. Article 21 of the Fundamental Rights deals with which subject.


(A) Protection of certain rights regarding freedom of speech,
(B) Protection in respect of conviction of offence
(C) Protection against arrest and detention in certain cases
(D) Protection of life and personal liberty

5. Who declares the financial emergency.


(A) President
(B) Prime Minister
(C) Finance Minister
(D) None of the above

6. After declaration of financial emergency by the President, what is the period of operation
without approval by the ParliamenT.
(A) Three Months
(B) Four Months
(C) Two Months
(D) One Month

7. Within what period, the Parliament has to approve Financial emergency declared by the
President
(A) Six Months
(B) Two Months
(C) Three Months
(D) Four Months

8. In Financial Emergency, salaries and allowances of which groups get reduction


(A) Central Government Employees
(B) Judges of the Supreme Court and High Courts
(C) A & B
(D) None of the above

9. Raj Pramukh represents whom under the Indian Constitution


(A) President
(B) Governor
(C) A & B
(D) None of the above

10. Under the Indian Constitution, what is the concept behind the protection of President and
Governors
(A) President & Governors are above the law
(B) President & Governors make any law for the Constitution
(C) President and Governors are answerable to Prime Minister
(D) President and Governors shall not be answerable to any court to the exercise and
performance of the powers and duties of their office

11. By which Constitutional amendment, the appellation Rajpramukh was omitted


(A) 7th Amendment Act 1956
(B) 4th Amendment Act 1955
(C) 6th Amendment Act 1956
(D) 10th Amendment Act 1961

12. Which constitutional article provides personal immunity for President and Governors for official
act
(A) Article 362
(B) Article 363
(C) Article 368
(D) Article 361

13. Which constitutional article provides personal immunity to the head of the states for his official
act from legal action, including proceedings for contempt of Court
(A) Article 361
(B) Article 362
(C) Article 368
(D) Article 369
14. Under which constitutional articles, newspapers do not have the right to publish report of the
proceedings of a secret session of either of the Houses of Parliament or Legislative Assembly &
Legislative Council
(A) 361
(B) 361A
(C) 361B
(C) 361C

15. Spell out the condition under Article 361A by which any person or newspaper cannot be sued
for legal proceeding if any report of proceedings of Parliament and State Legislature is
published
(A) The report must be a report of the `procedings' of a House of the Union or a State Legislature.
Hence, it must be relevant to a motion or other business before the House, an must not have
been expunged
(B) It must be a report as distinguished from one article or `Comment'.
(C) Such report must be substantially true. Hence, an extract or a garbed or perverted report
would not be protected. The reporting must not be actuated by malice
(D) All of the above .

16. Any Court including Supreme Court does not have constitutional right under Article 143 to
exercise jurisdiction over any dispute arising out of any provision of which agreements tha were in
operation before commencement of the Constitution
(A) Treaty, Agrrement
(B) Covenant, Engagement
(C) Sanad
(D) All of the above .

17. What is the meaning of Indian State in the Constitution


(A) Any territory recognised by President of India
(B) Any territory before commencement of Indian Constitution by the British ruler
(C) Any territory which government of the Dominion of India recognised
(D) B & C

18. Before which Constitutional Amendment, Prince, Chief or other person were recognised by
the President of India as the Ruler of the Indian State
(A) 26th Amendment Act 1971
(B) 24th Amendment Act 1971
(C) 16th Amendment Act 1963
(D) 17th Amendment Act 1964 .

19. Under which Constitutional Amendment Privy Purses were abolished


(A) 36th Amendment Act 1975
(B) 26th Amendment Act 1971
(C) 16th Amendment Act 1963
(D) 17th Amendment Act 1964

20. Under which Constitutional Article, Union Government has the power to give direction to
the State Govt. regarding any of the provisions of the Constitution
(A) Article 368
(B) Article 362
(C) Article 365
(D) Article 367

21. If any State Government fails to comply with or to give effect to any direction given by the
Union Government, who can come to conclusion that a situation has arisen in which the State
cannot carry out governance in accordance with the provision in the Constituion
(A) President
(B) Prime Minister
(C) Home Minister
(D) Supreme Court

22. Under Article 365 what are the duties of the Union Government with respect to State
Governments
(A) Ensure that every State Minister should act in accordance with the advice of Chief Minister
(B) Ensure that Governor acts under advice of the Chief Minister
(C) Ensure that Governance in the State is in accordance with the Constitution
(D) All of the above

23. What is the meaning of Foreign State as given in our Indian Constitution
(A) Federal State
(B) Commonwealth State
(C) Nation
(D) Any State other than India

24. Which Constitutional article defines the work of Administrative Tribunal


(A) Article 323A
(B) Article 233B
(C) Article 223B
(D) None of the above

25. Under which part of the Constitution, Tribunals have been defined
(A) Part Four
(B) Part Seven
(C) Part Fifteen
(D) Part Ten

26. What is the period laid down by the Constitution before the proposal for removal of
Speaker and Deputy Speaker of Lok Sabha can be taken up by a resolution in the Lok Sabha
(A) 15 Days
(B) 18 Days
(C) 16 Days
(D) 14 Days

27. In Lok Sabha, who can not preside in the House while a Resolution for Removal from his
office is under consideration
(A) Speaker
(B) Deputy Speaker
(C) A & B
(D) None of the above

28. Under which Article Salaries and allowances of the Chairman and Deputy Chairman of Rajya
Sabha and Speaker and Deputy Speaker of Lok Sabha are mentioned
(A) 97
(B) 96
(C) 95
(D) 94

29. Which Article mentions the conduct of business of the Houses of Parliament
(A) 99
(B) 100
(C) A & B
(D) None of the above

30. Who appoints each member of either of the Houses of the Parliament after notification is
received from the Election Commission
(A) President
(B) Speaker of Lok Sabha
(C) Chairman of Rajya Sabha
(D) Prime Minister

31. Who shall not give vote in the first instance in either of the Houses of Parliament
(A) Speaker
(B) Chairman
(C) A & B
(D) None of the above

32. When Speaker and Chairman shall give their votes on the Parliament.
(A) When Prime Minister asks them to give vote on the Bill
(B) When the House passes such a resolution
(C) In the case of a tie between Yes and No
(D) All the above

33. What is the Quorum laid down to constitute a meeting of either of the Houses of Parliament
(A) one-tenth of the total number of members of that House
(B) one-fourth of the total number of members of that House
(C) one-fifth of the total number of members of that House
(D) one-half of the total number of members of that House

34. Which Article mentions disqualification of members in the Parliament


(A) Article 101 to Article 104
(B) Article 101 to Articles 105
(C) Article 102 to Article 106
(D) Article 106 to Article 110

35. Lok Sabha has the supremacy in which matter


(A) Railway Budget
(B) Defence Budget
(C) Foreign affairs
(D) Financial Bill

36. Normally, what kind of session does the Parliament hold


(A) Budget session
(B) Monsoon session
(C) Winter session
(D) All the above

37. Which session of the year, President addresses both the Houses of Parliament
(A) First session (Budget)
(B) Second session (Monsoon)
(C) Third session (Winter)
(D) None of the above

38. In which session of Parliament, Railway and General Budgets are presented
(A) Monsoon session
(B) First session
(C) Winter session
(D) None of the above

39. What is the meaning of the adjournment motion under Parliamentary procedure
(A) Member draws attention regarding important subject-matter
(B) Member wants the House to discuss his subject-matter
(C) Member wants to raise complicated issue
(D) Member wants to draw the attention of the House to way recent matter of urgent public
importance having serious consequences.

40. Who has the power to accept adjournment in the House


(A) Prime Minister
(B) Home Minister
(C) Speaker in the Lok Sabha and Chairman in the Rajya Sabha
(D) All the above

41. Which authority in the Parliament has the right to adjourn the House
(A) Speaker of Lok Sabha and Chairman of Rajya Sabha
(B) President
(C) Parliamentary Affairs Minister
(D) Prime Minister

42. Who has the power to present adjournment motion in Lok Sabha and Rajya Sabha
(A) Minister
(B) Deputy Speaker
(C) Prime Minister
(D) Member of the said House

43. In the Parliament, every Bill has to pass through which stages of Reading before it becomes
act
(A) First Reading
(B) Second Reading
(C) Third Reading
(D) All the above
44. When a Bill is passed by the Parliament and the President, what is the status of the name
(A) Law
(B) Bill approved
(C) Bill exercised for administration
(D) Government procedure

45. Which two houses, can have a joint sitting


(A) Legislative Assembly and Parliament
(B) Lok Sabha and Rajya Sabha
(C) Council of State and Legislative Council
(D) None of the above

46. When does the President assent the Bill


(A) Lok Sabha passes the Bill
(B) Rajya Sabha passes the Bill
(C) Lok Sabha and Rajya Sabha both passed the Bill
(D) None of the above

47. In India, when does the financial year commence


(A) First April
(B) Second April
(C) First March
(D) Fifteenth March

48. On the subject of budget, demands for grant are arranged in which way
(A) Prime Minister
(B) Finance Minister
(C) Ministry wise
(D) All the above

49. In how many parts, the Budget is presented in Lok Sabha


(A) One
(B) Two
(C) Three
(D) Four

50. How are the parts of the Budget known as


(A) General Budget
(B) Railway Budget
(C) A & B
(D) None of the above

51. During T. N. Seshan period as Chief Election Commissioner, who was the other
Commissioner in the Election Commission
(A) M. S. Gill
(B) G. V. Krishnamurthy
(C) A & B
(D) None of the above

52. For which Election, one General Electoral Roll for every territorial Constituency shall exist
(A) Lok Sabha
(B) Rajya Sabha
(C) Legislature
(D) All the above

53. On what basis, Election to Lok Sabha and State Legislature shall be conducted
(A) Adult Suffrage
(B) Indirect Election
(C) Direct Election
(D) None of the above

54. Under which Constitutional Amendment, provision for minimum age as 18 years for the
Indian citizen was made to become eligible to vote
(A) 60th Amendment Act 1988
(B) 61st Amendment Act 1989
(C) 62nd Amendment Act 1989
(D) None of the above

55. Before 61st Amendment Act 1989, what was the age of Indian citizen eligible to vote in the
Election
(A) 23
(B) 24
(C) 21
(D) 22

56. Under Article 326, what was the Constitutional requirment for the Indian citizen not to become
eligible as a voter
(A) Non Resident
(B) Unsoundness of Mind
(C) Crime or Corrupt or Illegal practive
(D) All the above

57. Which Constitutional Article lays down qualification for becoming a voter
(A) Article 328
(B) Article 339
(C) Article 326
(D) Article 295

58. Which Constitutional Article lays down qualifications for the Indian citizens for election to
Parliament
(A) Article 81
(B) Article 80
(C) Article 83
(D) Article 84

59. Which Constitutional Article defines qualifications for the Indian citizen for election to a State
Legislature
(A) Article 173
(B) Article 175
(C) Article 177
(D) Article 178

60. Under the Indian Constitution, what does `Adult Suffrage' signify
(A) Children
(B) Persons
(C) Any Indian citizen who is of the age of 18 years and above
(D) None of the above

61. Who makes law with respect to Elections for State Legislature
(A) Parliament
(B) Judiciary
(C) Government
(D) Election Commission

62. Under Constitutional Articles 327 or 328, which subject shall not be called to be questioned
in any Court
(A) Delimitation of Constituencies
(B) The allotment of seats to such Constituency
(C) A & B
(D) None of the above
63. How the election to either House of Parliament or to either House of the Legislature of the
State shall be called in question in the courts whose manner of presentation may be provided
made by law by appropriate Legislature
(A) PIL (Public Interest Litigation)
(B) SLP (Special Leave Petition)
(C) Action under Article 32
(D) Election Petition

64. Which Constitutional Article lays down the reservation of seats for Scheduled Castes and
Scheduled Tribes in the Lok Sabha
(A) Article 330
(B) Article 332
(C) Article 333
(D) Article 334

65. Which Constitutional Article deals with `Representation of the Anglo-Indian Community' with
House of the People
(A) Article 334
(B) Article 331
(C) Article 332
(D) Article 333

66. Under Article 331, how many members of the Anglo-Indian Community can be nominated in
Lok Sabha by the President
(A) 3
(B) 4
(C) 2
(D) 1

67. In which State a separate district has been reserved for Scheduled Tribes
(A) Assam
(B) Andhra Pradesh
(C) Karnataka
(D) Kerala

68. Which Constitutional Article deals with representation of the Anglo-Indian Community in the
Legislative Assembly
(A) Article 334
(B) Article 335
(C) Article 336
(D) Article 333

69. Under Article 333, how many members from the Anglo-Indian Community can be
nominated by the Governor in the Legislative Assembly
(A) 8
(B) 1
(C) 4
(D) 3

70. Under which Constitutional Amendment of Article 334, reservation of seats for Scheduled
Castes and Scheduled Tribes in the Lok Sabha and State Legislative Assembly has been laid
down
(A) 31st Amendment Act 1959
(B) 23rd Amendment Act
(C) 45th Amendment Act 1980 & 62nd Amendment Act 1989
(D) All the above

71. Before which Constitutional Amendment, 20 years were fixed for reservation of seats for
Scheduled Castes and Scheduled Tribes in Lok Sabha and State Legislature
(A) 23rd Amendment Act 1969
(B) 8th Amendment Act 1959
(C) 44th Amendment Act 1978
(D) 45th Amendment Act 1980
72. Through which Constitutional Amendment, 30 years were fixed for reservation of seats for
SC and ST in Lok Sabha and State Assembly
(A) 45th Amendment Act 1980
(B) 50th Amendment Act 1984
(C) 23rd Amendment Act 1969
(D) 51st Amendment Act 1984

73. Which Constitutional Amendment fixes 40 years for reservation of seats for SC and ST in Lok
Sabha and State Legislative Assembly.
(A) 55th Amendment Act 1986
(B) 56th Amendment Act 1987
(C) 52nd Amendment Act 1985
(D) 45th Amendment Act 1980

74. Which Constitutional Amendment lays down 50 years for reservation of seats for SC and ST
in the House of People and State Legislative Assembly
(A) 62nd Amendment Act 1989
(B) 44th Amendment Act 1928
(C) 45th Amendment Act 1980
(D) None of the above

75. Which Constitutional Article mentions Claims of SC and ST in Govt. services and
post
(A) Article 336
(B) Article 335
(B) Article 338
(D) Article 339

76. In the Parliament, what is the meaning of the Government Bill


(A) Bill presented by Ruling Party member
(B) Bill approved by the Government
(C) Only the Prime Minister presents the Bill
(D) A Bill introduced by any Minister in either of the Houses of the Parliament

77. In the Second Reading, what kind of process is adopted to approve the Bill
(A) A general discussion on the Bill
(B) Clause by clause consideration of the Bill
(C) A & B
(D) None of the above

78. Who has the authority to call a joint sitting of the two Houses of Parliament
(A) Prime Minister
(B) President
(C) Member of Lok Sabha
(D) Member of Rajya Sabha

79. Who has the power to accord his assent or withhold his assent to a Bill passed by the
parliament
(A) President
(B) Member of the House
(C) Minister
(D) None of the above

80. Which Bill President can neither return nor withhold his assent
(A) Defence Bill
(B) Money Bill
(C) Law Bill
(D) Financial Account Committee Bill

81. How may Standing Committees are there in Lok Sabha


(A) 14
(B) 15
(C) 15
(D) 18 .

82. Standing Committees in Lok Sabha are


(A) Business Advisory Committee & Committee of Privileges
(B) Committee on Absence of Members from the sitting of the House & Committee on
Estimates
(C) Committee on Government assurances and Committee on papers laid on the Table
(D) All the above

83. Financial Committees in Lok Sabha are


(A) Committee on Estimates
(B) Public Accounts Committee
(C) Public Undertaking Committee
(D) All the above

84. Ad hoc Committees in Parliament are


(A) Committee on Draft Five Year Plan, etc.
(B) Committee in the conduct of certain members during the President Address (C) Select or
Joint Committee on Bills
(D) All the above

85. Who appoints Ad hoc Committee on Parliament


(A) Speaker of Lok Sabha
(B) Chairman of Rajya Sabha
(C) A & B
(D) None of the above

86. By which procedure the Indian President and American President are elected as laid down
by their country's constitution
(A) Elected through Member of Legislature
(B) Elected by the People
(C) Elected by State Legislatures
(D) Elected by an Electoral College

87. In what way our Indian Parliament is not Sovereign or Supreme with respect to the
Constitution
(A) In the Preamble, Constitution of India defines people of India as Sovereign
(B) Written Constitution of India
(C) Separation of Power and Checks and Blanees between the three constitutional organ
(D) All the above

88. Who has said that basic features of the Indian Constitution do not amount to a change
(A) Prime Minister
(B) Parliament
(C) Supreme Court of India
(D) Government .

89. What is the nature of India's political system


(A) Presidential System
(B) Parliamentary System
(C) A & B
(D) None of the above

90. Which Constitutional Article was very much affected in the Supreme Court Judgement of
Kesavanand Bharti vs. State of Kerala
(A) Article 352
(B) Article 368
(C) Article 351
(D) Article 342

91. Which constitutional article emopowers amendment in the Constitution of India


(A) Article 368
(B) Article 356
(C) Article 357
(D) Article 359

92. Which constitutional organ has the power to amend Constitution of India
(A) Judiciary
(B) Executive
(C) Legislative
(D) Parliament

93. On which subject, Parliament has the power to amend the Constitution and the same also
need ratification by the State Legislature
(A) Articles 54, 55, 73, 162 and 241 or Chapter IV of Part V, Chapter V of Part VI or Chapter I of
Part XI
(B) Any of the Lists in the Seventh Schedules of the representation of State on Parliament
(C) The Provisions of Article 368
(D) All the above

94. Under which Constitutional Amendment Act, Article 368 of the Constitution was amended
for the first time
(A) 25th Amendment Act
(B) 26th Amendment Act
(C) 24th Amendment Act
(D) 27th Amendment Act

95. Which Supreme Court Judgement pronounced that Fundamental Rights cannot be
abridged
(A) Golak Nath vs. State of Punjab A.I.R. 1967 S.C. 1643
(B) Kesavanand Bharti vs. State of Kerala A.I.R. 1973 S.C. 1961
(C) Indira Gandhi vs. Rajnarain A.I.R. 1975 S.C. 2299
(D) None of the above

96. Who curbed the Judicial Review power of Judiciary through Amendment of theConstitution?
(A) State Legislature
(B) Parliament
(C) Council of State
(D) Legislative Council

97. Who restored the Judicial Review power of Judiciary under Indian Constitution?
(A) Supreme Court of India
(B) High Court
(C) Chief Metropolitan Magistrate
(D) District Court

98. In which House, Janta Government failed to secure two-third majority for new clause under
Article 368 for introducing referendum for effecting changes in certain logic features of the
Constitution?
(A) Legislative Council
(B) State Legislature
(C) State Assembly
(D) Rajya Sabha

99. Who said in his judgement that no part of our Constitution is unamendable-
(A) Allahabad High Court
(B) Calcutta High Court
(C) Madras High Court
(D) Supreme Court of India

100. What was the important landmark judgement regarding amendment of the Constitution
(Article 368)
(A) Shankari Prasad vs. Union of India
(B) Golak Nath vs State of Punjab
(C) Kesavananda vs State of Kerala, Minerva Mill vs. Union of India
(D) All the above

Answers :
1.(A) 2.(C) 3.(B) 4.(D) 5.(A) 6.(C) 7.(B) 8.(C) 9.(C) 10.(D)
11.(A) 12.(D) 13.(A) 14.(B) 15.(D) 16.(D) 17.(D) 18.(A) 19.(B) 20.(C)
21.(A) 22.(C) 23.(D) 24.(A) 25.(C) 26.(D) 27.(C) 28.(A) 29.(C) 30.(A)
31.(C) 32.(C) 33.(A) 34.(A) 35.(D) 36.(D) 37.(A) 38.(B) 39.(D) 40.(C)
41.(A) 42.(D) 43.(D) 44.(A) 45.(B) 46.(C) 47.(A) 48.(C) 49.(B) 50.(C)
51.(C) 52.(D) 53.(A) 54.(B) 55.(C) 56.(D) 57.(C) 58.(D) 59.(A) 60.(C)
61.(A) 62.(C) 63.(D) 64.(A) 65.(B) 66.(C) 67.(A) 68.(D) 69.(B) 70.(D)
71.(B) 72.(C) 73.(D) 74.(A) 75.(B) 76.(D) 77.(C) 78.(B) 79.(A) 80.(B)
81.(D) 82.(D) 83.(D) 84.(D) 85.(C) 86.(D) 87.(D) 88.(C) 89.(B) 90.(B)
91.(A) 92.(D) 93.(D) 94.(C) 95.(A) 96.(B) 97.(A) 98.(D) 99.(D) 100.(D)

Q.1 When was Indian Independence Act passed, by whom and on the basis
of what plan?

Ans. Thee Indian Independence Act was passed by the British Parliament on 5th
July in 1947. This act was passed to give effect to the Mountbatten plan creating the
two independent states of India and Pakistan.

Q.2- What were the options before the Indian native states in the Indian
Independence Act?

Ans. The Indian Independence Act gave three options to the Indian native states
(1) to join India(2) to join Pakistan(3) to remain Independent.

Q.3 -Who drafted the Indian Constitution?

Ans. The Indian constitution was drafted by the Drafting Committee of the
Constituent Assembly.

Q.4 -Who was the chairman of the Constituent Assembly? Who was
the Chairman of the Drafting Committee?
Ans. Dr. Rajendra Prasad was the Chairman of Constituent Assembly. Dr. B.R.
Ambedkar was the Chairman of Drafting Committee.

Q.5 -When was the Constitution adopted?

Ans. The constitution was adopted on 26th November 1949.

Q.6 -When did the Constitution come into force?

Ans. The constitution came into force on 26th January 1950.

Q.7 -According to the Preamble India is what kind of state?

Ans. According to the original preamble India is a Sovereign, Democratic Republic.


By 42nd amendment of the constitution India is made also a Socialist and Secular
state.
Q.8 -What are the political ideals, according to the Preamble India seeks to
secure?

Ans. India seeks to secure to her people:

Justice: Social Economic and Political.

Liberty: of thought, expression, belief, faith and worship

Equality: of status and opportunity, and Fraternity, assuring the dignity of the
individual and unity of the nation.

Q.9 -How many Arts, are there in the Indian Constitution?

Ans. The original constitution had 395 Arts and 7 schedules. Now after the 80th
amendment of the constitution there are 405 Arts and 10 schedules in the
constitution.

Q.10 -What is meant by Secular and Socialist as described in the Preamble?

Ans. The preamble describes India to be a secular state. It means that there is no
established religion in India and that state does not give any preference to any
religion in India.

The 42nd amendment of the Indian constitution makes India a socialist state. It
means that the state shall abolish private ownership of the means of production and
distribution. This however has not yet been achieved. On the other hand the state
now encourages private ownership.

Q.11 Bring out the significance of the terms Sovereign, Democratic


Republic as mentioned in the Preamble.

Ans. India is a sovereign state. It means the state in India is the supreme authority
over all men and all associations within the country and is absolutely free from any
outside control.

India is democratic. It means that in India all governments are formed on the basis of
popular support.

India is republic. It means all offices of the state from the highest to the lowest are
held on the basis of merit and no office of the state is held on the basis of hereditary
right.

Q.1 2- How many states and union territories are there in the Indian Union?

Ans. There are 25 states and 7 union territories in the Indian Union.

Q.13 Which State of the Indian Union has a separate Constitution?

Ans. The state of Jammu and Kashmir has a separate constitution.


Q.14 What is the importance of Arts 370 of the Constitution?

Ans. This Art gives the state of Jammu & Kashmir a separate constitutional status.
Laws passed by the Indian Parliament apply to Jammu & Kashmir if they are
accepted by the J. K. legislature.

Q.15 Is the preamble to the Constitution justiciable in character?

Ans. No. The preamble is not justiciable.

Q.16 What is the necessity of the Preamble?

Ans. The preamble is like an introduction to the constitution. The courts use the
preamble to clarify: the meaning of the Law whenever there is an ambiguity.

Q.17 Does the Indian Constitution recognize the principle of dual citizenship?

Ans. Though India is a federation, the principle of dual citizenship has not been
accepted in the constitution. All Indians are accepted as the citizens of India. There is
no citizenship of the states.

Q.18 When was the Indian Citizenship Act passed?

Ans. Indian Citizenship Act was passed in 1955.

Q.19 Into how many categories are the Indian citizens divided?

Ans. Indian citizens are divided into two categoriescitizens by birth and citizens
by adoption.

Q.20 What are the provisions of the Indian Constitution regarding integration
or creation of new states?

Ans. The Indian Parliament by Acts passed that it can integrate new states into
India or can create new states out of the territory of an existing state or states.

Q.21 How many categories of fundamental rights of citizens has been


recognized by the Indian Constitution?

Ans. Originally the constitution conferred on the Indian citizens seven fundamental
rights. They are (a) right to equality (b) right to freedom (c) right against exploitation
(d) right to religious freedom. (e) right to education and culture (f) right to property and
(g) right to constitutional remedies:. At present there are six fundamental rights. Right
to private property has been removed from the list of fundamental rights by the 44th
amendment of the constitution.

Q.22 What is meant by equality in the eye of law?

Ans. Equality in the eye of law as provided by Art. 14 of the constitution mean that
nobody is above the law. Law applies to all persons equally.
Q.23 What are the freedoms granted to citizens by Art 19 of the Constitution?

Ans. Art 19 of the constitution grants six freedoms to the citizens They are right to
freedom of

(I) expression, (2) to assemble peaceably and without arms (3) to form association,
(4) to move freely in India, (5) to five anywhere in India and (6) to adopt any
profession or business.

Q.24 What Art of the Constitution forbids use of titles or honor conferred by
foreign states?

Ans. Art 18.

Q.25 Is the right to work and employment recognized by Constitution?

Ans. No. The directive principle of the constitution declares right to work and
employment as desirable. But the directives are non- justiciable in character.

Q.26 Are the conferment of honour like Bharat Ratna or Padma Bibhusun
violative of right to equality under Art. 114?

Ans. No, because Bharat Ratna, Padma Bibhusun etc. are not considered as titles.
They cannot be used before or after the names of the persons on whom they are
conferred.

Q.27 How the Indian Constitution seeks to protect children against


exploitation?

Ans. Arts 24 of the constitution forbids employment of children in factories, mines,


or in hazardous works.

Q.28 What categories of people can be imprisoned without trial?

Ans. Under Arts 22 (3) of the constitution enemy aliens and persons arrested under
preventive detention Acts can be imprisoned without trial.

Q.29 India is a Secular State. What does it mean?

Ans. Indian secularism means that the state does not have any established religion,
that people are free to practice and profess any religion and that the state does not
show any preference to any religion.

Q.30 -What Arts of the Constitutions confer right to freedom of religion?

Ans. Arts 25 to 28 confer right to freedom of religion on the citizens.


Q.31 Is possession of private property a Fundamental Right?

Ans. Private property was a fundamental right before the passing of 44th
amendment of the constitution. Now the possession of private property is an ordinary
legal right and not a fundamental right.

Q.32 -What Art of the Constitution confers right to constitutional remedies?

Ans. Art 32 of the constitution confer the right to constitutional remedies on the
citizens.

Q.33 How many kinds of writs the Supreme Court or the High Court may
issue in case of transgression of Fundamental Rights?

Ans. The Supreme Court or the High Courts may issue five kinds of writs.

These are writs of (1) Habeas Corpus (2) Mandamus (3) Prohibition (4) Certiorari and
(5) Quowarranto.

Q.34 What is Habeas Corpus? What Art of the Constitution provides for it?

Ans. Habeas Corpus: literally means that human person is sacred. Hence no man
can be detained illegally. Whenever a man is detained he must be produced before a
court. This writ is a powerful safeguard against arbitrary arrest and detention Art. 32
of the constitution provides for Habeas Corpus.

Q.35 -Do the members of the armed forces enjoy the Fundamental Rights
given by the Constitution?

Ans. The parliament may restrict the fundamental rights by passing laws. Beyond
such restrictions the members of the Armed Forces enjoy their fundamental rights.

Q.36 -What are principal duties of the Indian Citizens?

Ans. Obeying the constitution, showing respect to the national flag and the national
anthem, defending Indias sovereignty, integrity, and unity protecting national
properties and upholding Indias glorious mixed culture and also showing respect to
woman are the principal duties of the Indian citizens.

Q.37 -When were the duties of the citizens added to the Constitution?

Ans. Ten duties of the Indian citizens have been added to the constitution by the
42nd amendment to the constitution.

Q.38 In what chapter of the Constitution and in what Arts are the Directive
principles of the Constitution given?

Ans. In chapter IV Arts 35-51 the directive principle of the constitution are given.
Q.39 -What is the principal difference between the directive principles and the
fundamental rights?

Ans. The fundamental rights are justiciable while the directive principals are non-
justiciable i.e. the fundamental rights are enforced by the courts while the directives
are not enforced by the courts.

Q.40 -Name four important directives given in the Indian Constitution?

Ans. (1) The states should provide help in cases of old age, unemployment and
disability. (2) State should strive to reduce inequality between individuals, groups and
professions (3) State should promote and foster rural cottage industries. (4) The state
should provide compulsory free primary education to children below 14 years if age.

Q.41 Name two directives based on the ideal of socialism?

Ans. (A) The state should prevent concentration of the ownership of the means of
production in the hands of the few (Art. 39c).

(B) The state should provide help and assistance in case of unemployment and
disability.

Q.42 -What directive is based on a Gandhian ideal?

Ans. -Art 45 of the constitution directs the state to promote and foster Panchayeti Raj
in India.

Q.43 -What are the value and importance of the directive principles?

Ans. The directive principles though non-justiciable are not worthless. Sir B.N. Rao
contends that the directives are moral precepts. K.M. Panikkar holds that the
directives promise India to achieve economic socialism or economic democracy.

Q.44 -What directive aims to secure separation of the executive from the
judiciary?

Ans. Art 50 of the constitution direct the state to separate the executive from the
judiciary. This is important to secure the independence of the judiciary from executive
control and influence.

Q.45 -How the Arts related to fundamental rights may be amended?

Ans. The Arts related to fundamental rights may be amended when a bill to that
effect is passed by 2/3 votes of the members present and voting in each House of the
Parliament but the members present and voting must constitute a majority of total
membership of each House.

Q.46 In what Art of the Indian Constitution India has been described as a
Union of States.

Ans. Art one of the Indian constitutions says that India that is Bharat shall be a
Union of States.
Q.47 What is the other name of India given in the Constitution and in what
Art.?

Ans. India is also called Bharat in Art.1 of the constitution.

Q.48 Does any Art of the Indian Constitution use the term federation? If not
what is the term used in the constitution by which India can be called a
federation?

Ans. No Art of the constitution calls India a federation. Instead Art.1 of the
constitution calls India a Union of States. From this, constitutional experts infer that
India is a federation.

Q.49 An indestructible federation of indestructible states Does this


description apply to India?

Ans. President Abraham Lincoln called the United States an indestructible


federation of indestructible states. This description does not apply to India because
an Indian state may be easily destroyed through the process of Re-organization of
states. Hence India may be called an indestructible federation of very much
destructible state.

Q.50 Through how many lists powers have been distributed between the
Union and the States in India? Name them.

Ans. The Indian constitution distributes powers between the union and the states
through three lists, the Union list, the State list and the Concurrent list.

Q.51 What is the procedure through which powers have been distributed
between the Union and the States in India?

Ans. The Indian Constitution has not followed either the U. S. or the Canadian
system of distribution of powers between the federal government and the
governments of the federating units. In the U. S. A. there is only one list of powers of
the national government. The residues belong to the states. In Canada there is only
one list of powers of the governments of the provinces and the residues belong to the
central government. In India there are three liststhe Union list, the State list and the
Concurrent list. The residues belong to the centre.

Q. 52 Under what circumstances the union government can legislate on


subjects in the state list?

Ans. On three occasions the Union government may legislate on state subjects.(i)
to give effect to an international treaty, (ii) when the Rajyasabha by 2/3 majority
authorized the Parliament under Art 249 to legislate on any state subject, (iii) when
one or more states request the union government to legislate on any state subject.
Q. 53 Under what Art the President may constitute Inter State Councils and
for what purpose?

Ans. Under Arts 263 of the Indian constitution the President may constitute Inter
State Councils to regulate the relations between the centre and the states or between
the states.

Q. 54 Name four important subjects in the Union list?

Ans. Defence, Foreign relations, Citizenship and Banking are four important
subjects in the Union list.

Q. 55 Name four important subjects in State list?

Ans. (i) Land Revenue, (ii) Law and order, (iii) Local government, (iv) Education up
to the secondary level.

Q. 56 What do you mean by Concurrent list? Name four important subjects in


Concurrent list.

Ans. The concurrent list mentions the subjects on which the union and the state
governments, both can legislate but in case of conflict between a union and state law
the union law prevails. Administrative and criminal law, vagrancy, forests, protection
of wild animals and birds are four important subjects in the concurrent list.

Q. 57 Mention two circumstances when the Union Government can issue


directives to the State Governments.

Ans. The union government may issue directive to the state government for the
protection of the Railways and Ports and also on subjects considered important in the
national interest.

Q. 58 What is meant by residual powers?

Ans. In a federal constitution powers not mentioned in any list of powers given to
the centre or the states is known as the residual power. In India the residuary belongs
to the union government.

Q. 59 Mention two taxes the proceeds of which are divided between the
union and the state governments.

Ans. Non-agricultural income tax and excise duties except on cosmetics and
medicines.

Q. 60 Mention four subjects on which the state government may impose


taxes.

Ans. The state government may realize (i) sales tax, (ii) land revenue, (iii)
amusement tax and taxes on (iv) transportations.
1. The largest constitution in the world ?
a.Australia
b.Japan
c.Indian
d.Canada

Answer

c.Indian

2. Indian Constitution was adopted by the Government of India on :


a.26 Jan 1950
b.26 Nov 1949
c.26 Dec 1950
d.15 Aug 1948

Answer

b.26 Nov 1949

3. How many articles are there in Indian Constitution originally


a.448
b.385
c.495
d.395

Answer

d.395

4. Constitution of India is divided into 12 schedules. true or false?


a.True
b.False

Answer

a.True

5. Equality before law is mentioned in which article of the Indian Constitution?


a.14
b.10
c.9
d.17

Answer

a.14

6. Which article contains the right to freedom of speech ?


a.Article 17
b.Article 11
c.Article 19
d.Article 5

Answer

c.Article 19

7. Which article deals with Right to elementary Education ?


a.Article 21A
b.Article 31D
c.Article 15
d.Article 39A

Answer

a.Article 21A

8. Fundamental Rights are granted to citizens under which Article ?


a.Article 36 51
b.Article 5 11
c.Article 52 151
d.Article 12 35

Answer

d.Article 12 35

9. Article 17 of the Indian Constitution deals with ?


a.Abolition of titles
b.Abolition of Untouchability
c.Equality before law
d.Right to education

Answer

b.Abolition of Untouchability

10. Protection of interst of minorities is mentioned in which article?


a.Article 18
b.Article 4
c.Article 29
d.Article 25

Answer

c.Article 29

11. Fundamental Duties is mentioned in which article of the Indian Constitution?


a.Article 43A
b.Article 48A
c.Article 21A
d.Article 51A

Answer

d.Article 51A

12. Which part of the Indian constitution deals with the directive principles?
a.Part IV
b.Part VII
c.Part IX
d.Part III

Answer

a.Part IV

13. How many schedules are there in Indian constitution?


a.15
b.10
c.35
d.12

Answer

d.12

14. In which article of the Indian Constitution the admission or establishment of new states is
mentioned?
a.Article 1
b.Article 2
c.Article 3
d.Article 4

Answer

b.Article 2

15. Rights of citizenship of certain persons who have migrated to India from Pakistan is
mentioned in which article ?
a.Article 4
b.Article 8
c.Article 6
d.Article 10

Answer

c.Article 6

16. Who made the statement The preamble of the constitution is the horoscope of
Indian Republic ?
a.K. M. Munshi
b.Jawaharlal Nehru
c.B.R.Ambedkar
d.Mahatma Gandhi

Answer

a.K. M. Munshi

17. Parliament to regulate the right of citizenship by law is mentioned in which article of the
Indian Constitution ?
a.Article 41
b.Article 37
c.Article 13
d.Article 11

Answer

d.Article 11

18. Article 24 of the Indian Constitution deals with ?


a.Right to education
b.Protection of life and personal liberty
c.Prohibition of employment of children in factories
d.Prohibition of traffic in human beings and forced labour

Answer

c.Prohibition of employment of children in factories


19. Provision for free and compulsory education for children is mentioned in which articlea.
a.Article 35
b.Article 45
c.Article 27
d.Article 55

Answer

b.Article 45

20. Procedure for the impeachment of the President is mentioned in which article of the Indian
Constitution?
a.Article 59
b.Article 71
c.Article 140
d.Article 61

Answer

d.Article 61

21. Article 148 of the Indian Constitution deals with


a.Rules of court
b.Comptroller and Auditor General of India
c.Seat of Suprme Court
d.Appropriation of Bills

Answer

b.Comptroller and Auditor General of India

22. Annual financial statement is mentioned in which article of the Indian Constitution?
a.Article 112
b.Article 125
c.Article 140
d.Article 131

Answer

a.Article 112

23. Establishment and constitution of Supreme court mentioned in which article


a.Article 142
b.Article 88
c.Article 124
d.Article 138

Answer

c.Article 124

24. Duties and powers of the comptroller and auditor general is mentioned in which article?
a.Article 137
b.Article 149
c.Article 98
d.Article 150

Answer

b.Article 149
25. Who is known as the chief architect of Indian Constitution ?
a.Jawaharlal Nehru
b.Mahatma Gandhi
c.RajendraPrasad
d.Dr.B.R. Ambedkar

Answer

d.Dr.B.R. Ambedkar
1. Who is regarded as the chief architect of the Indian Constitution?

Ans:- B.R.Ambedkar

2. When the Constitution of India came into effect on?

Ans:- 26th Jan, 1950

3. 26th January was chosen as Republic day to commemorate

Ans:- The Purna Swaraj Declaration in 1930

4. Which day is observed as Republic Day every year?

Ans:- 26th January

Q. How long earlier the World War II had ended before the Indian Constitution came into effect in
1950?

Answer : World War II had ended 5 years earlier in 1945

5. Who was the chairman of the Minorities Committee?

Ans:- Harendra Coomar Mukerjee

6. Who was elected the president of Constituent Assembly after Sacchidananda Sinha?

Ans:- Dr. Rajendra Prasad

7. What are the two major languages, the original constitution written in?

Ans:- The original constitution written in Hindi and English

8. Which most important feature made Indian Constitution unique to any constitution of the world?

Answer : Indian constitution is the only longest written constitution in the world ever

9. When Hindi became the Official Language of India?

Answer : The Indian Constitution declared Hindi as the official language in 1950

10. When English became the official language of India across Hindi?

Answer : According to Official Language Act 1963, English became official language of India in
1965 along with Hindi

11. What was the basic structure of the Indian constitution during the time of commencement?

Ans:- During the time of commencement there were 395 articles, 22 parts and 8
schedules. Present structure is 448 articles, 12 schedules, 5 appendices and 100
ammendments till 2015

12. Who is the constitutional head of the Indian union?

Answer : President of India


13. Constitutionally who is the second powerful person in India?

Answer : Vice-President

Q. One of the basic character of the Indian Constitution is

Answer : Judicial Review

Q. What is the major basic feature of the Indian Constitution?

Answer : Fundamental rights

14. When the Sarkaria Commission was set up?

Answer : 1983

15. How the Sarkaria Commission is related to Indian Constitution?

Answer : Sarkaria Commission can supervise the required changes of the constitution and the
balance of power between state and central governments. Ranjit Singh Sarkaria is the head of
this commission

16. Directive principles of state policy in Indian Constitution has been adopted from which
country?

Ans:- Ireland

Q. What is the objective of Directive Principles of State Policy of the constititution?

Answer : Directive Principles are the guideline to making laws by the government. In part IV of the
Indian Constitution, these provisions can be enforced by the courts.

17. What is the constitutional name of India?

Answer : Republic of India

18. Which famous bengali artists have a great contribution in designing of Indian constitution?

Ans:- Nandalal Bose including Beohar Rammanohar Sinha

Constitutional facts : It is notable that Indian Constitution is influenced by the constitution of


different nations. As India is large and densely populated country so it took too much time to
finish the original constitution. An expenditure of Rs 1 Crore spent and almost 5 Years took on this
project.

19. The fundamental rights feature borrowed from which nation?

Ans:- United States of America

Q. Who said this famous constitutional quote that Law and order are the medicine of the body
politic and when the body politic gets sick, medicine must be administered.

Answer : Dr. Bhim Rao Ambedkar

20. Liberty, Equality and Fraternity are borrowed features of which nationality?

Ans:- French Constitution

21. Distribution of power between Union and state govt policy borrowed from which nation?

Ans:- Canada

22. The idea of Single Citizenship taken from which country?

Ans:- Britain
23. By which legislation India and Pakistan were divided as two independent nations?

Answer : Indian Independence Act 1947

24. Currently there are how many fundamental rights are recognized by the Indian constitution?

Answer : There are six fundamental rights- 1) Right to equality 2) Right to freedom 3) Right
against exploitation 4) Right to freedom of religion 5) Cultural and educational rights 6) Right to
constitutional remedies

25. Indian constitutional rights were inspired by which historical documents?

Answer : Frances Declaration of the Rights of Man and United States Bill of Rights

26. Why 26 November is observed as Indian Constitution Day or Samvidhan Divas?

Answer : Constitution Day which is also known as Samvidhan Divas is celebrated on 26


November each year to commemorate the adoption of Indian Constitution. On this day in 1949 the
Constituent Assembly of India adopted the Constitution of India that came into effect on 26
January 1950. Government of India declared 26 November as Constitution Day on November
2015 on a occassion of B. R. Ambedkars birth anniversary.

27. Who is empowered by the Indian Constitution to appoint the Attorney General of India?

Answer : President of India

28. Who is the only person holding the right of appointment of Chief Justice of India?

Answer : President of India

29. By which procedure the President of India may be removed before the expiry of the term?

Answer : Impeachment

30. Who appoints the Chief Election Commissioner of India?

Answer : President of India

31. Who appoints Vice President of India?

Answer : The Electoral College of India

32. Who is the inaugural Prime Minister of India?

Answer : Jawaharlal Nehru(1947-1964)

33. Who is the first Deputy Prime Minister of India?

Answer : Sardar Vallabhbhai Patel(1947-1950)

34. Who is the first President of India?

Answer : Dr. Rajendra Prasad

Q. Who is the famous writer of Introduction to the Constitution of India?

Answer : Durga Das Basu

Q. Who said this famous quote that Constitution is not a mere lawyers document, it is a vehicle of
Life, and its spirit is always the spirit of Age.?

Answer : Dr. Bhim Rao Ambedkar

35. Who was the British Prime Minister during Indias Independence?

Answer : Clement Atlee


36. What was the Mountbatten Plan?

Answer : The Mountbatten Plan was an agreement between Lord Mountbatten and Indian
National Congress, Muslim League and Sikh community

37. Who was the Governor General during Indias Independence?

Answer : Lord Mountbatten

38. Who is empowered to appoint the Judges of Supreme Court?

Answer : President on advice of the Chief Justice

39. Who appoints the members of Council of Ministers?

Answer : President on advice of the Prime Minister

40. Who appoints the Comptroller and Auditor General(CAG)?

Answer : President appoints the Comptroller and Auditor General of India

41. Who appoints the Chairman and other members of the UPSC(Union Public Service
Commission)?

Answer : President

42. Who can proclaim a Financial Emergency?

Answer : President can proclaim a financial emergency

43. How long the National Emergency can be imposed?

Answer : Six months. May be extended by six month on Parliamentary approval.

Preamble, Constituent Assembly, Drafting Quiz


44. When the Preamble of Constitution of India adopted by the constituent assembly?

Ans:- 26th Nov, 1949

45. How long it took to drafting the Constitution of India?

Ans:- 3 years

46. When the last session of constituent assembly was held?

Answer : 24 January 1950

47. Who was the legal advisor to the constitutions Drafting Committee?

Answer : B. N. Rau

48. Under which commission the first constituent assembly election was held?

Answer : Cabinet Mission Plan 1946

49. How many members were there in Constituent Assembly?

Answer : 389

50. Constituent Assembly is based on which structure?

Answer : Unicameral

51. Who was the President of All India Muslim League?


Answer : Muhammad Ali Jinnah

52. How the members of Constituent Assembly were elected by?

Answer : By a single transferable votting system

53. Who was the first temporary two day president of the Constituent Assembly?

Ans:- Sachchidananda Sinha

54. On 29th August 1947 how many members were appointed by drafting committee and who
was the chairman?

Ans:- B.R.Ambedkar(chairman of Drafting Committee) and members assisted by a


constitutional advisor. Members were Pandit Govind Ballav Pant, Kanaiyalal Maneklal Munshi,
Alladi Krishnaswamy Iyer, N.Gopalaswamy Ayenger, B.L.Mitter, Md. Saadullah and D.P. Khaitan

55. How many times and when the preamble has been amended so far?

Answer : So far the preamble has been amended only once. On 18 December 1976 during the
time of Emergency in India in Prime Ministership of Indira Gandhi

56. What was the significance of August Offer?

Answer : On August 1940 a proposal offered by Viceroy Lord Linlithgow about the expansion of
Executive Council, is called August Offer.

57. When the idea of Constituent Assembly was proposed?

Answer : In the year 1934 by M. N. Roy

58. Which was the major political party having maximum seats in Constituent Assembly?

Answer : Congress

59. What was the old name before India became Republic in 1950?

Answer : Dominion of India

60. Who was the President of Provincial Constitution Committee of Constituent Assembly?

Answer : Sardar Vallabhbhai Patel

61. Who was the President of States Committee of Constituent Assembly?

Answer : Jawaharlal Nehru

Constitutional Ammendment, Articles, Parts,Schedules Quiz


62. Which part of constitution is related to statutory povisions for Panchayet Raj in villages?

Answer : Part 9

Q. By which constitutional amendment the goods and services tax(GST) adopted in India from 1
July 2017?

Answer : 122nd Amendment

63. Which article is related to provisions for setting up Rent Controls Tribunals?

Answer : Article 323B

64. Which article is related to reservation of SC/ST Employees in promotion?

Answer : Article 16
Q. By which article, practice of untouchability has been declared a punishable offence by law?

Answer : Article 17

Q. Which article prohibits state from conferring any titles other than military or academic
distinctions?

Answer : Article 18

Q. Which article ensures six freedoms as civil rights to Indian citizens?

Answer : Article 19 ensures six freedoms which include- Freedom of speech and expression,
Freedom of association, Freedom of assembly without arms, Freedom of movement, Freedom to
practise any profession and Freedom to reside in any part of the Indian union.

Q. Human trafficking and forced labour are the punishable offence by law under which article?

Answer : Article 23

Q. Which article prohibits employment of child labour under the age of 14?

Answer : Article 24

65. Which part of constitution contains the Directive Principles of State Policy?

Answer : Part IV

66. In the Indian Constitution article 45 deals with which issue?

Answer : Article 45 ensures Provision for free and compulsory education for children

67. In the Indian Constitution article 48-A deals with which issue?

Answer : Article 48-A ensures Protection and improvement of environment and safeguarding of
forests and wildlife

68. The Fundamental Duties are inspired by which country?

Answer : USSR (Now Russia)

69. Which article states that constitution is the supreme law of the nation?

Answer : Article 13

70. Which articles of Indian constitution provides the fundamental right against exploitation?

Answer : Article 23 and 24 of Indian constitution

71. Attorney General of India is appointed by which article of Indian Constitution?

Answer : Article 76(1)

72. Who is empowered by Indian Constitution to employ a state Chief Minister?

Answer : Governor

73. By which article President can ask the Attorney General of India to attend Parliamentary
sessions and to report him?

Answer : Article 88

74. By which article President can dismiss a state Governor?

Answer : Article 156

75. By which article of Indian Constitution President carry the power of pardoning?
Answer : Article 72

76. By which article Financial Emergency is proclaimed?

Answer : Article 360

77. By which article President of India can declare a National Emergency?

Answer : Article 352

78. Which Part of Constitution indicates the relation between States and Union territories of India?

Answer : Part I

79. The Election Commission of India operates under which article of Indian Constitution?

Answer : Article 324

80. By which ammendment Socialist and Secular two words have been included in Indian
Constitution?

Ans:- 42nd Ammendment in 1976

81. By which article the Constituion of India can be ammended?

Ans:- Article 368, Part XX

82. What is the significance behind 61st amendment of article 326 in 1989?

Answer : Reducing the age limit for voting rights from 21 to 18

83. By which constitutional amendment Sikkim became a state of Indian Union in 1975?

Answer : 36th Amendment

84. What is the features of 15th amendment in 1963?

Answer : Raising retirement age of High Court judges from 60 to 62

85. By which constitutional amendment and when Nagaland was formed as a state of Indian
Union?

Answer : 13th Amendment in 1963

86. By which constitutional amendment and when Right to Property was excluded from
fundamental rights list?

Answer : 44th Amendment in 1978

87. By which constitutional amendment article 31-C inserted into the Directive Principles of State
Policy?

Answer : 25th Amendment Act of 1971

88. By which amendment the Fundamental Duties of citizens were added to the Indian
Constitution?

Answer : 42nd Amendment in 1976

89. Sindhi was included as an official language by amending which schedule in 1967?

Answer : 8th schedule

90. By which constitutional amendment Konkani, Manipuri and Nepali included as Official
Languages?
Answer : Amending of schedule 8 by 71st amendment in 1992

91. Which article amendment is related to reservation for Scheduled Tribes in Tripura State
Legislative Assembly?

Answer : Amendment of article 332 in 1992

92. Which part of constitution indicates the relation between the Union and States?

Ans:- Part XI

93. Which article indicates to protect, preserve and defend law and constitution by President?

Ans:- Article 60

94. Which article ensures the Prime Ministers leadership in Union Cabinet of India?

Ans:- Article 74

95. Which article of Indian Constitution detaches Jammu & Kashmir state from India?

Ans:- Article 370

Presidents Rule Quiz


96. When the Presidents rule first imposed in West Bengal?

Answer : 1962

97. When the Presidents rule first imposed in Uttarakhand?

Answer : 27 March, 2016

98. How the constitution empowers by the 68th amendment of article 356 in 1991?

Answer : Article 356 amended to permit Presidents rule up to five years in the state of Punjab.

99. Which article of Indian constitution refers to composition of Presidents rule?

Answer : Article 356

100. When the article 365 was used for the first time in India?

Answer : Year 1954

101. What is the first Presidents rule imposed state in India?

Answer : Kerala, in 1959

102. How long the Presidents rule can be continued as per constitution?

Answer : 3 Years on approval of Parliament in every 6 months.

103. When the Presidents rule was imposed for the first time in Karnataka?

Answer : Year 1971

104. When the Presidents rule first imposed in Uttar Pradesh?

Answer : 25 February, 1968

105. When the Presidents rule first imposed in Tripura?

Answer : 1 November, 1971

106. When the Presidents rule first imposed in Tamilnadu?


Answer : 31 January 1976

107. When the Presidents rule first imposed in Sikkim?

Answer : 18 August 1978

108. When the Presidents rule first imposed in Punjab?

Answer : 1951

109. When the Presidents rule first imposed in Odisha?

Answer : 25 February 1961

Parliamentary System GK Quiz


110. Which one & only body has the right to amend the Constitution of India?

Answer : Parliament

111. Money Bill must be originated first in which body of Parliament?

Answer : Lok Sabha

Q. What is the only constitutional body has the supreme power of making laws?

Answer : Parliament of India is the supreme legislative body where laws are made as per Indian
Constitution

112. Within how many days Money Bill can be recommended by council of states?

Answer : 14 Days

113. There are how many type of sub-committees in Parliamentary committee?

Answer : Two- 1) Ad hoc Committee 2) Standing Committee

114. Who is the Chairman of the Union Cabinet?

Answer : Prime Minister is the chairman of Union Cabinet

115. Union Cabinet is collectively responsible to which house of Parliament?

Answer : Lok Sabha

116. Parliamentary form of govt policy has been taken from which country?

Ans:- British Constitution

117. Which body of Parliament can be dissolved by the President?

a) Lok Sabha

b) Rajya Sabha

118. How many members are appointed by the President to the Rajya Sabha?

Answer : 12 Members(Specially qualified in different field like art, science or literature)

119. Who elects the two nominees from Anglo-Indian Community in Lok Sabha?

Answer : Two by President

120. A bill passed in Parliament cant become a law without permission of-

Answer : President
121. In Parliamentary System how many seats there in Lok Sabha?

Answer : 545 Seats

122. In Parliamentary System how many seats are there in Rajya Sabha?

Answer : 245 Seats

123. How many sessions are conducted by Parliament each year?

Answer : Three sessions a) Budget session b) Monsoon session c) Winter session

124. Who is the current Chairman of Rajya Sabha as of 2017?

Answer : Mohammad Hamid Ansari(vice-president)

125. Who is the Deputy Chairman of the Rajya Sabha as of 2017?

Answer : P. J. Kurien

126. Who is the current Speaker of the Lok Sabha as of 2017?

Answer : Sumitra Mahajan

127. Who is the current Deputy Speaker of the Lok Sabha as of 2017?

Answer : M. Thambidurai

128. Who is the leader of the lower house Lok Sabha?

Answer : Narendra Modi (Prime Minister)BJP

129. Who is the current leader of the upper house Rajya Sabha?

Answer : Arun Jaitley

130. How the Rajya Sabha MPs are elected by?

Answer : Rajya Sabha MPs(Member of Parliament) are elected by the members of All the State
Legislative Assembly

131. How the Lok Sabha MPs are elected ?

Answer : The Lok Sabha MPs are directly elected by Indian public voting through Lok Sabha
Election

132. Who designed the Parliament House(Sansad Bhavan) in New Delhi?

Answer : Edwin Lutyens and Herbert Baker

133. Who is the Head of two house of Parliament?

Answer : President

134. What is the age requirement to become a member of Lok Sabha?

Answer : 25 Years

Judiciary Gk Quiz
135. Who is the chief adviser to the President of India?

Answer : Prime Minister

136. Judicial review is the borrowed feature of which country?


Answer : United States of America

137. Judicial review is related to which article?

Answer : Article 13

138. What is the retirement age limit of a judge in Supreme Court?

Answer : 65 Years is the maximum age limit of retirement

139. Who is the current Chief Justice of India as of 2017?

Answer : Jagdish Singh Khehar is the current Chief Justice of India?

140. When the Supreme Court of India came in to force?

Answer : The Supreme Court of India came into force on 28 January 1950

141. When the Indian High Courts Act was enacted?

Answer : In the year 1861 Indian High Courts Act was enacted to create High Courts

142. When the Federal Court of India replaced by Supreme Court of India?

Answer : Until the establishment of Supreme Court of India in 1950

143. Which judicial body worked under Government of India act?

Answer : Federal Court of India worked under Government of India Act 1935

1. The council of Ministers of the National Capital Territory of Delhi shall consist of not
more than
Ans : 10 Ministers

2. District Judges in a State are appointed by the


Ans : Governor

3. According to the provisions of the Constitution, the administrative control(posting,


promotion, leave etc.) over the district courts and other subordinate courts, shall be
vested in the
Ans : High Court

4. If a stay order has been granted by the High Court in a case, the application of the
affected party by such an order shall be disposed of by the High Court within a period of
Ans : Two week

5. The power to make rules for the transaction of the business of the Government of
Indian and for the allocation of business among Ministers lies with the
Ans : President

6. Which Union territory send their members to the Council of States?


Ans : Pondicherry and Delhi

7. According to Article 243 of the Constitution, the Panchayats at the intermediate level
may not be constituted in a State which has a population
Ans : Not exceeding 20 lakhs
8.The power to determine the number of judges in a High Court lies with the
Ans : President of India

9. The Point of Order in a parliamentary practice involves


Ans : The question of rules of procedure not followed in a particular matter

10. How many members are nominated by the Governor in the Legislative Council of a
State?
Ans : 1/12 of the total membership

11. What is the maximum limit of total membership of the Legislative Council of a State?
Ans : 1/3 of the total membership of the Legislative Assembly

12. Which of the following categories of members are represented in the Legislative
Council of a State?
Ans : Representatives of Municipalities, Representatives of teachers, Representatives of
graduates

13. The members of the Legislative Council appointed by the Governor shall have special
knowledge or practical experience in respect of such matters as
Ans : Literature, Science, Art, Co-operative movement and Social Service

14. In which State in India, is a certain amount of grants paid out of the Consolidated
Fund of the State for the maintenance of some Hindu temples and shrines, as per the
provisions of the Constitution?
Ans : Tamilnadu

15. The President of India can issue a proclamation of National Emergency only on the
written recommendation of
Ans : The Cabinet consisting of only Cabinet Ministers of the Union

16. Under the provisions of Article 343 of the Constitution, Hindi was declared the official
language of the Union. But the use of English language was allowed to continue for the
official purpose of the Union for a period of
Ans : 15 years

17. Which category of minority have been recognized by the Constitution of India?
Ans : Religious minorities and Linguistic minorities

18 How many times has the proclamation of National Emergency been enforced in India?
Ans : Three times

19. At present, how many members of the Lok Sabha can be elected from Union
Territories?
Ans : 20
20 The Comptroller and Auditor-General of India is removed from his office by
Ans : The President on the basis of a resolution passed by both the Houses of Parliament
by special majority

21. Which schedule of the Constitution make special provisions for the administration and
control of scheduled Areas and Scheduled Tribes?
Ans : Fifth Schedule

22. Which schedule of the Constitution makes provisions for the administration of Tribal
Areas of Assam, Meghalaya, Tripura and Mizoram?
Ans : Sixth Schedule

23. A major portion of the Constitution of India is derived from the


Ans : Government of India Act, 1935

24. A Bill passed by the Legislative Assembly of a State can be delayed by the Legislative
Council for a maximum period of?
Ans : Three month

25. According to the provisions of Article 243, which category of person do not have
mandatory reservations of seats in Panchayats?
Ans : Backward Classes

26.The minimum age prescribed for election as a member of panchayat is


Ans : 21 years

Directive Principle of State Policy (DPSP)


Directive Principle of State Policy provides guidelines to central & state
government in India, to be kept in mind while framing laws & policies &
mentioned in part 4 of the constitution.
DPSP + FR >> Conscience of Indian Constitution

Basic aim of DPSPs is to set up social & economic goals before the law
makers
To bring socio-economic change in the country
To fulfill the basic needs of the common man
To reshape the structure of Indian society in direction of greater socio-
economic equality.

DPSPs are fundamentals in governance of the country & shall be considered


dutifully by the state while making laws, but DPSPs are not enforceable in
court of law
If state fails to fulfill these obligations, one cannot go to court of law
DPSPs only provides a yardstick for measuring success or failure of the
government

Articles 36 to 51 deal with the provisions of the Directive Principles & are
broadly classified into
Socialist principles

Gandhian principles
Liberal intellectual principles

Socialist Principles
To secure a social order for the promotion of welfare of the people.
To strive to minimise inequalities of income i.e. operation of the
economic system does not result in the concentration of wealth and
means of production to the common detriment;
ownership and control of the material resources of the community are
so distributed as best to subserve the common good;
Equal justice and free legal aid.
Ownership and control of material resources of the community shall be
so distributed so as to subserve the common good.
Equal pay for equal work.
Health & strength of workers, and the tender age of children must not
be abused.
Right to work, to education and to public assistance in certain cases.
Provision of just and humane conditions for work and maternity relief.
Participation of workers in the management of the industries.
Duty of the State to raise the level of nutrition and the standard of
living and to improve public health.
Children are given opportunities and facilities to develop in a healthy
manner and in conditions of freedom and dignity and that childhood
and youth are protected against exploitation and against moral and
material abandonment.
The Western Liberal Principles
Uniform Civil Code for the citizens.
Provide free and compulsory education for children below 14 years.
Separation of Judiciary from Executive.
To promote international peace and amity.
Protection of monuments and places and objects of national
importance
Protection and improvement of environment and safeguarding of
forests and wild life.

The Gandhian Principles


Organization of Village Panchayats & to promote cottage industry.
Promotion of educational and economic interests of the SCs, the STs
and the other weaker sections of the society.
To bring about the prohibition of intoxicating drinks and drugs that are
injurious to health.
Organization of agriculture and animal husbandry on modern and
scientific lines to prohibit the slaughter of cows, calves and other milch
and draught animals.

97th Amendment: Promotion of cooperative societies

Directives in other parts of the Constitution (Except part IV)


Article 350 A: It enjoins every State and every local authority within the
State to provide adequate facilities for the instructions in the mother tongue at
the primary stage to children of linguistic minority areas.
Article 351: It enjoins the Union to promote the spread of Hindi Language
so that it may serve as a medium of expression of all the elements of the
composite culture of India.
Article 335: It says that the claims of SC/ST shall be taken into
consideration, consistently with the maintenance of efficiency of
administration, in the making of appointments to services and posts in
connection with affairs of the Union or of a State.
Under the implementation of DPSP, Zamindari, Jagirdari & inamdari systems
were abolished & actual tillers of the soil were made owner of the land.
DPSPs v/s FRs
DPSPs spell out the character of
FRs provide the foundation of social & economic democracy in
political democracy in India India

Lays down negative obligation / DPSPs are positive / moral


restriction on the state i.e. FRs are obligations of state towards the
prohibitive in nature citizens

FRs have been laid down in clear DPSPs are laid in general terms
legal language in constitution and are sort of moral obligations

FRs represent something static i.e. DPSPs represents a dynamic


to preserve certain rights which move towards the betterment of
already exist the citizens

DPSPs are non-justifiable in


FRs are justifiable in nature & can nature & can not be enforced in
be enforced in court of law court of law

What if laws made by state giving effect to DPSPs violates FR, can
they be valid? This question was raised before SC in 1951 in
Champakam Dorairajan case !
SC held that they are supplementary to each other & no as such inherent conflict is between
them, thus, as far as possible, should be interpreted harmoniously. However, if it is not
possible, FRs will prevail over DPSPs means DPSPs can not override FRs, hence such a law is
void.

On this ground, SC held Bank nationalisation act & Privy purse (abolition) act unconstitutional

In 1971, 25th amendment introduced a new article 31-c, which states that if state enacts any
law giving effect to two directive principles viz. Equitable distribution of wealth (article 39-b)
& Prevention of concentration of wealth in fewer hands (article 39-c), & in that process if the
law violates FRs (article 14, 19 & 31), it can not be held void merely on this ground.
Article 31-c further states that such a law giving effect to 39 b & 39 c, can not be
questioned in court of law.
25th amendment was challenged in Kesavananda Bharati case (1973), in which SC held that
only first part of the article 31-c (overriding article 14, 19 & 31) is valid, but second part
which bars judicial review held unconstitutional.
42nd amendment, 1976 further amended article 31-c & widened its scope & gave precedence
for all DPSPs over article 14, 19 & 31 & hence made them immune to judicial review.
Minerva Mills case, 1980: SC struck down the changes introduced by 42ndamendment in
article 31-c & held them unconstitutional on grounds that total exclusion of judicial review
would offend the basic structure of the constitution.

Indian Polity and Governance


Constitution Historical Background, Making of the Constitution, Philosophy of the
Constitution
Focus:

Historical background various Acts passed from Regulation Act, 1773 to Indian
Independence Act, 1947.
Give more importance to Acts post 1857, when British Crown assumed sovereignty
over India from the East India Company.
Be aware of at least two-three major provisions of an Act, and how did it help in creating
a basic framework for the governance or administration during the British rule.

Example:

Indian-Councils Act, 1909 For the first time direct election was introduced; It provided
for a separate electorate for the Muslims

Making of the Constitution Context in which the Constituent Assembly was framed.
Role of Cripps Mission in the making of the Constitution
Composition and role of the Constituent Assembly and its criticism
Salient Features of the Constitution
Schedules and Sources from where it has been borrowed

Philosophy of the Constitution The Preamble -its importance and utility; Understand
the objective of the Preamble along with the key words like Sovereignty, Socialist,
Secular, Justice etc.

Example (2012 Prelims Questions Paper):

The distribution of powers between the Centre and the


States in the Indian Constitution is based on the scheme
provided in the(a) Morley-Minto Reforms, 1909(b) Montagu-
Chelmsford Act, 1919

(c) Government of India Act, 1935


(d) Indian Independence Act, 1947
Solution (c)

Example (2013 Prelims Questions Paper):


Economic Justice the objectives of Constitution has been
as one of the Indian provided in:(a) the Preamble and
Fundamental Rights

(b) the Preamble and the Directive Principles of State Policy


(c) the Fundamental Rights and the Directive Principles of State
Policy
(d) None of the above
Solution (b)

Example (2014 Prelims Questions Paper):

Consider the following statements : A Constitutional


Government is one which1. Places effective restrictions on
individual liberty in the interest of State Authority

2. Places effective restrictions on the Authority of the State in the


interest of individual liberty
Which of the statements given above is / are correct?
A. 1 only
B. 2 only.
C. Both 1 and 2
D. Neither 1 nor 2
Solution (b)

Which one of the following Schedules of the Constitution of


India contains provisions regarding anti-
defection?A. Second Schedule

B. Fifth Schedule
C. Eighth Schedule
D. Tenth Schedule
Solution (d)

Union and its Territory


Focus:

Article 1, Article 2, Article 3 and Article 4 how article 2 is different from Article 3?
Understand the expression Union of India, how its different from the Territory of the
India
Parliaments power to reorganize State.
Different Commissions set-up for Reorganization of States DHAR commission and
JVP commission, Fazal Ali commission

Note: You dont have to remember formation of each State. Just remember the
1st State which was formed based on the Fazal Ali Commission and the recently
formed State- Telangana
Citizenship
Focus:

Constitutional Rights and Privileges of Citizens of India


Acquisition and termination of citizenship
Difference between a PIO, NRI, OCI
Any recent policies and schemes related to PIO,NRI or OCI
For example, Visa on Arrival Scheme

Note: Make note on the recent changes made regarding Citizenship, voting rights. Pre-
Independence provisions can be skipped

Fundamental Rights (FR) [ Part III ]and Fundamental Duties[ Part IV-A ]
Focus
Fundamental Rights-

Its significance implications and applications are important, than just remembering all
the articles. Have a broader understanding of classification of Fundamental Rights and
to which section of people are they targeted.
Example: Article 29, 30, 31 protects the Cultural and Educational Rights of Minorities.
FR available to citizens of India only and both to citizens and foreigners
Difference between Fundamental rights and Rights secured by other provisions of the
Constitution like Writs
Different types of Writs (Article 32) and its scope
Present position of Right to Property
Exceptional cases with regard to FRs Armed forces and when Martial Law is enforced

Fundamental Duties

Committee recommendations like Swaran Singh, Verma Committee


General awareness on list of FDs
Enforcement of FDs is it justiciable or non-justiciable
Significance and Criticism of FDs

Example (2011 Prelims Questions Paper):

In India, if a religious sect/community is given the status of


a national minority, what special advantages it is entitled
to?1. It can establish and administer exclusive educational
institutions.2. The President of India automatically nominates a
representative of the community to LokSabha.

3. It can derive benefits from the Prime Ministers 15-Point


Programme.
Which of the statements given above is/are correct?(a) 1 only
(b) 2 and 3 only
(c) 1 and 3 only
(d) 1, 2 and 3
Solution (c)

Under the Constitution of India, which one of the following is


not a fundamental duty?(a) To vote in public elections(b) To
develop the scientific temper

(c) To safeguard public property


(d) To abide by me Constitution and respect its ideals
Solution (a)

Example (2012 Prelims Questions Paper

Which of the following is/are among the Fundamental Duties


of citizens laid down in the Indian Constitution?1. To
preserve the rich heritage of our composite culture2. To protect
the weaker sections from social injustice

3. To develop the scientific temper and spirit of inquiry


4. To strive towards excellence in all spheres of individual and
collective activity
Select the correct answer using the codes given below :
(a) 1 and 2 only
(b) 2 only
(c) 1, 3 and 4 only
(d) 1, 2, 3 and 4
Solution (c)

Directive Principles of State Policy (DPSP) [ Part IV ]


Focus:

Classification of DPSP- Socialistic, Gandhian, Liberal-Intellectual principles


Its significance & Principle- like socio-economic aspect
Justifiable or not- enforceable in court or not
Comparison between DPSP and Fundamental Rights,
Conflicts between them Chronology of cases from Champalam Dorairajan case
(1951) to Minerva Mills case (1980) and the present position of FRs w.r.t DPSP
Significance of 42nd and 44th Amendment Act w.r.t DPSP

Note: Give special importance to Golaknath Case, Keshavananda Bharthi, Minerva


Mills, Maneka Gandhi case, Berubari Union, 3 Judges case.

Example (2013 Prelims Questions Paper):

According to the Constitution of India, which of the following


are fundamental for the governance of the country?(a)
Fundamental Rights(b) Fundamental Duties
(c) Directive Principles of State Policy
(d) Fundamental Rights and Fundamental Duties
Solution (c)

Example (2014 Prelims Questions Paper):

In the Constitution of India, promotion of international peace


and security is included in theA. Preamble to the
ConstitutionB. Directive Principles of State Policy (DPSP)

C. Fundamental Duties
D. Ninth Schedule
Solution (b)

Amendment of the Constitution


Focus

Procedure for amendment based on Simple and Special majority (Article 368);
provisions of the Constitution which can be amended based on Simple and Special
majority.
Amendability of Fundamenta Rights
What constitutes Basic Structure like Supremacy of the Constitution, Judicial Review,
Secular character etc.
Important Constitutional Amendments Acts 42nd, 44th , 61st 72nd, 73rd
Criticism

Note: Note down the recent amendments from 91st Amendment Act, 2003 till date.
Example (2013 Prelims Questions Paper):

Consider the following statements1. An amendment to the


Constitution of India can be initiated by an introduction of a bill in
the LokSabha only.2. If such an amendment seeks to make
changes in the federal character of the Constitution, the
amendment also requires to be ratified by the legislature of all the
States of India.

Which of the statements given above is/are correct?


(a) 1 only
(b) 2 only
(c) Both 1 and 2
(d) Neither 1 nor 2
Solution (d)

Union Government

A general understanding on the difference between Prime Ministerial and Presidential


form of Government, its merits and de-merits.
Union Executive
President, Vice-President, Prime Minister, Central Council of Ministers

The executive institutions should be studied on with the following points in


Focus:

Election procedure/Mode of Appointment


Powers and Functions Example: Veto power, Ordinance making power, Pardoning
power (article 72) of the president
Resignation/Removal/Impeachment
Relationship between President and Prime-Minister with regard to their roles and
responsibilities

Note: Be aware of the recent issues w.r.t to Pardoning power (as was in the case of
Ajmal Kasab, Afzal Guru), Impeachment, Ordinance-making Power of the President
Central Council of Ministers (COM)

Importance of 91st Amendment Act with regard to composition of COM


Difference between COMs and Cabinet Ministers
Cabinet ministers, Cabinet Committees their role and responsibilities
Understand the concept of collective responsibility, kitchen cabinet

Example (2012 Prelims Questions Paper):

The Prime Minister of India, at the time of his/her


appointment: (2012)(a) need not necessarily be a member of
one of the Houses of the Parliament but must become a member
of one of the Houses within six months(b) need not necessarily
be a member of one of the Houses of the Parliament but must
become a member of the LokSabha within six months

(c) must be a member of one of the Houses of the Parliament


(d) must be a member of the LokSabha
Solution (a)

Example (2013 Prelims Questions Paper):

Consider the following statements:1. The Council of Ministers


in the Centre shall be collectively responsible to the Parliament.2.
The Union Ministers shall hold the office during the pleasure of
the President of India.

3. The Prime Minister shall communicate to the President about


the proposals for legislation.
Which of the Statements given above is/are correct?
(a) 1 only
(b) 2 and 3 only
(c) 1 and 3 only
(d) 1,2 and 3
Solution (b)

Example (2014 Prelims Questions Paper):

Consider the following statements:1. The President shall


make rules for the more convenient transaction of the business of
the Government of India, and for the allocation among Ministers
of the said business.2. All executive actions of the Government
of India shall be expressed to be taken in the name of the Prime
Minister.

Which of the statements given above is / are correct?


A. 1 Only
B. 2 only
C. Both 1 and 2
D. Neither 1 nor 2
Solution (a)

Union Legislature Parliament


Atleast 25-30% (out of total questions in Polity) of the marks come from this
chapter and is the most important topic from the exams perspective. The entire
chapter is important!
Focus:

Working of the Parliament-


Role and functions of the Parliament
Sessions, Motions, Parliamentary procedure Summoning, Prorogation, Joint sitting
Devices of Parliamentary proceedings like Question Hour, Zero Hour, Adjournment
Motion etc.
Similarities and Differences between Lok Sabha, Rajya Sabha, their composition,
election, powers and functions; Special powers of Rajya Sabha
Qualifications and Disqualifications of members; Anti defection law
Parliamentary Privileges
Enactment/Procedure to make Law-
Different stages in passing of a Bill
Different types of bills Ordinary Bill, Money Bill, Finance Bill its importance
Budget-
Enactment of Budget
What does the budget contain? Its significance
What do you understand by Charged Expenditure?
Funds available like Consolidated Fund, Contingency Fund, Public Accounts Fund-
who authorizes it, what does it contain?
Role of important institutions in the Parliament like Speaker and Chairman of
RajyaSabha their election, removal/resignation, functions
Role of Leader of opposition (LoP)
Importance of Parliamentary Committees Estimates Committee, Business Advisory
Committee, Committee on Government Assurance their composition, functions, and
a general idea about the less significant committees like Rules Committee etc.

Example (2011 Prelims Questions Paper):

What is the difference between vote-on-account and


interim budget?(2011)1. The provision of a vote-on-account
is used by a regular Government, while an interim budget is a
provision used by a caretaker Government.2. A vote-on-
account only deals with the expenditure in Governments
budget, while an interim budget includes both expenditure and
receipts.

Which of the statements; given above is/are correct?


(a) 1 only
(b) 2 only
(c) Both 1 and 2
(d) Neither 1 nor 2
Solution (b)

Example (2014 Prelims Questions Paper):

Which one of the following is the largest Committee of the


Parliament?A. The Committee on Public AccountsB. The
Committee on Estimates

C. The Committee on Public Undertakings


D. The Committee on Petitions
Solution (b)

Consider the following statements regarding a No-Confidence Motion in


India:1. There is no mention of a No-Confidence Motion in the Constitution of India.2. A
Motion of No-Confidence can be introduced in the Lok Sabha only.

Which of the statements given above is / are correct?


A. 1 only
B. 2 only
C. Both 1 and 2
D. Neither 1 nor 2
Answer: (c)
1) For election to the Lok Sabha, a nomination paper can be filed by

a) anyone residing in India

b) a resident of the constituency from which the election is to be


contested

c) any citizen of India whose name appears in the electoral roll of a


constituency

d) any citizen of India

Why not option a? in that case even an alien residing in India could contest the
election

Why not option b? explanation give in the image below

Why not option c? any citizen includes even children & others who are not
electors(whose names dont appear on the electoral roll)

2) In the context of Indian History, the principle of Dyarchy refers to

a) Division of the central legislature into two houses

b) Introduction of double government i.e., Central and State governments

c) Having two sets of rulers; one in London and another in Delhi

d) Division of the subjects delegated to the provinces into two categories


3) Consider the following statements

1. In the election for Lok Sabha or State Assembly, the winning candidate
must get at least 50 percent of the votes polled, to be declared elected

2. According to the provisions laid down in the Constitution of India, in


Lok Sabha, the Speakers post goes to the majority party and the Deputy
Speakers to the opposition

Which of the statements given above is/are correct?

a) 1 only

b) 2 only

c) Both 1 & 2

d) Neither 1 nor 2

About statement 1 candidate is not required to get at least 50 percent of the


votes, he just needs to secure maximum number of votes among all

About statement 2 Speaker will be from majority party & Deputy Speaker from
opposition is a convention not a constitutional provision
4) Right to vote and to be elected in India is a

a) Fundamental Right

b) Natural Right

c) Constitutional Right

d) Legal Right

(Edited : Find discussions in the next blog)

5) Consider the following statements

1. The Election Commission of India is a five-member body

2. Union Ministry of Home Affairs decides the election schedule for the
conduct of both general elections and bye-elections

3. Election Commission resolves the disputes relating to splits/mergers of


recognised political parties

Which of the statements given above is/are correct?

a) 1 & 2 only

b) 2 only

c) 2 & 3 only

d) 3 only

(Please note that for this question in the original blog, answer given was C. It was
by mistake. Though it could be inferred from the explanation given below,
correction has been made)

It is a known fact that Election Commission of India is a three-member body.


Now for the remaining two statements, we dont find the details in Laxmikanth. The
same details can be found on the official website of Election Commission of
India.(Images given below)

6) In India, Judicial Review implies


a) the power of the judiciary to pronounce upon the constitutionality of laws and
executive orders

b) the power of the judiciary to question the wisdom of the laws enacted
by the legislatures

c) the power of the judiciary to review all the legislative enactments before
they are assented to by the President

d) the power of the judiciary to review its own judgements given earlier in
similar or different cases

6.

7) Which of the following are not necessarily the consequences of the


proclamation of the Presidents Rule in a State?

1. Dissolution of the State Legislative Assembly

2. Removal of the Council of Ministers in the State

3. Dissolution of the local bodies

Select the correct answer using the code given below

a) 1 & 2 only

b) 1 & 3 only

c) 2 & 3 only

d) 1, 2 & 3
It needs to be focused that it is not required for the President to dissolve the State
Legislative Assembly in the event of proclamation of President Rule, he may
either suspend or dissolve it. So we can conclude that 1 is not necessarily the
consequence of the proclamation of Presidents Rule.

There is no mention about local bodies in case of proclamation of local bodies, so


again that too is not necessarily the consequence of the proclamation of
Presidents Rule.

8) Which of the following are envisaged by the Right against Exploitation in


the Constitution of India?

1. Prohibition of traffic in human beings and forced labour

2. Abolition of untouchablity

3. Protection of the interests of minorities

4. Prohibition of employment of children in factories and mines

Select the correct answer using the code given below

a) 1, 2 & 4 only

b) 2, 3 & 4 only

c) 1 & 4 only

d) 1, 2, 3 & 4
Abolition of Untouchability is a part of Right to Equality.

Protection of the interests of minorities is a part of Cultural & Educational Rights.

9) The main advantage of Parliamentary form of government is that

a) the executive and legislature work independently

b) it provides continuity of policy and is more efficient

c) the executive remains responsible to the legislature

d) the head of the government cannot be changed without election


10) Out of the following statements, choose the one that brings out the principle
underlying the Cabinet form of Government

a) An arrangement for minimizing the criticism against the Government whose


responsibilities are complex and hard to carry out to the satisfaction of all

b) A mechanism for speeding up the activities of the Government whose


responsibilities are increasing day by day

c) A mechanism of Parliamentary democracy for ensuring collective


responsibility of the Government to the people

d) A device for strengthening the hands of the head of the Government whose
hold over the people is in a state of decline

As highlighted above, parliamentary form government is also known as cabinet


form of government.
11) Which one of the following is not a feature of Indian Federalism?

a) There is an independent judiciary in India

b) Powers have been clearly divided between the Centre and the States

c) The federating units have been given unequal representation in the


Rajya Sabha

d) It is the result of an arrangement among the federating units

We do have independent judiciary, we do have division of powers between the


Centre & the States in schedule-7 of the Constitution and the federating units have
been given unequal representation in Rajya Sabha(unlike in the Senate in USA).
12) Which of the following statement/s is/are true of the Fundamental Duties
of an Indian Citizen?

1. A legislative process has been provided for the enforcement of these


duties

2. They are correlative to legal duties

Select the correct answer using the code given below

a) 1 only

b) 2 only

c) Both 1 & 2

d) Neither 1 nor 2

There is no legislative process provided for the enforcement of Fundamental


duties in the Constitution. Constitution just mentions duties, no process for their
enforcement.

Second statement has been discussed in great detail in the classroom(not


possible to discuss here for now, if possible will narrate it in a separate blog)

13) Which one of the following objectives is not embodied in the Preamble to
the Constitution of India?

a) Liberty of Thought

b) Economic Liberty

c) Liberty of Expression

d) Liberty of Belief

Students must recall the recitation of the Preamble we used to do in the


classroom. I hope everyone would have it by heart.
14) Democracys superior virtue lies in the fact that it calls into activity

a) the intelligence and character of ordinary men and women

b) the methods for strengthening executive leadership

c) a superior individual with dynamism and vision

d) a band of dedicated party workers

An easy conclusion.

15) In the context of India, which one of the following is the correct
relationship between Rights and Duties?

a) Rights are correlative with Duties

b) Rights are personal and hence independent of society and Duties


c) Rights, not Duties, are important for the advancement of the
personality of the citizen

d) Duties, not Rights, are important for the stability of the State

While studying theory of duties, we come to know & realize how closely they are
connected with the rights.(Discussed in great detail in the classroom).

Rights cannot be divorced from duties, nor duties can be divorced from the Rights.

16) The mind of the makers of the Constitution of India is reflected in which
of the following?

a) The Preamble

b) The Fundamental Rights

c) The Directive Principles of State Policy

d) The Fundamental Duties

Again Preamble, an easy conclusion.


17) The Parliament of India exercises control over the functions of the
Council of Minister through?

1. Adjournment motion

2. Question hour

3. Supplementary questions

Select the correct answer using the code given below

a) 1 only

b) 2 & 3 only

c) 1 & 3 only

d) 1, 2 & 3

A student of Public Administration would find it easier to answer(Chapter


Accountability & Control).

All the three devices are there for the legislative control over the executive.

18) With reference to the Parliament of India, consider the following


statements

1. A private members bill is a bill presented by a Member of Parliament


who is not elected but only nominated by the President of India

2. Recently, a private members bill has been passed in the Parliament of


India for the first time in its history
Which of the statements given above is/are correct?

a) 1 only

b) 2 only

c) Both 1 & 2

d) Neither 1 nor 2

Private member is private member whether elected or nominated. So first


statement is wrong. There is no such distinction as regards bills being presented
by them.

Below I have given the image from the book called Our Parliament by Subhash
Kashyap to prove that second statement too is incorrect.

19) One of the implications of equality in society is the absence of

a) Privileges

b) Restraints

c) Competition

d) Ideology
20) Which principle among the following was added to the Directive
Principles of State Policy by the 42nd Amendment to the Constitution?

a) Equal pay for equal work for both men and women

b) Participation of workers in the management of industries

c) Right to work, education and public assistance

d) Securing living wage and human conditions of work to workers


21) Which one of the following statements is correct?

a) Rights are claims of the State against the citizens

b) Rights are privileges which are incorporated in the Constitution of a


State

c) Rights are claims of the citizens against the State

d) Rights are privileges of a few citizens against the many

RIGHTS ARE CLAIMS.

The question I liked the most as we had repeatedly discussed this topic in the
class. What could be more beautiful!

One needs to read Political Science NCERT in detail in order to answer such
questions.

22) Local Self-government can be best explained as an exercise in


a) Federalism

b) Democratic Decentralisation

c) Administrative Delegation

d) Direct Democracy

23) Consider the following statements

With reference to the Constitution of India, the Directive Principles of


State Policy constitute limitations upon

1. Legislative function

2. Executive function

Which of the above statement/s is/are correct?

a) 1 only

b) 2 only

c) Both 1 & 2

d) Neither 1 nor 2

DPSP are in the nature of instructions or recommendations to state(legislature as


well as executive) they cannot be said to be limitations upon the states action.

Some Fundamental Rights can be said to be limitations upon the action of the
state not DPSP. Thats why it is said that Fundamental Rights are negative in
nature whereas DPSP are positive in nature.

Вам также может понравиться